A company sells 1000 packs of cards per day at a price of $5 per pack. With every $0.02 reduction in price, 10 more packs a day are sold. Under these conditions, what is the maximum possible income per day, and what price per pack of cards will produce this income? Include how much extra money is made with this new price structure.

Answers

Answer 1
Explanation

From the statement, we know that:

0. the company sells 1000 packs of cards per day for $5 per pack,

,

1. with every $0.02 reduction in price, 10 more packs a day are sold.

To solve this problem, we define the following variables and functions:

• r = # of price reductions,

• n(r) = # of packs sold by the company as a function of r,

,

• p(r) = price per pack (in $) as a function of the # of price reductions,

,

• I(r) = income (in $) as a function of the # of price reductions.

Using points 1 and 2, we write the following functions: function of # of packs sold n(r) as:

• the i

[tex]n(r)=1000+10*r,[/tex]

• the function of the price per pack p(r):

[tex]p(r)=5-0.02*r,[/tex]

• the function for the income I(r) is given by the product of the # of packs sold n(r) and the price per pack p(r):

[tex]I(r)=n(r)*p(r)=(1000+10*r)*(5-0.02*r)=-0.2r^2+30r+5000.[/tex]

(1) To find the maximum income, we must maximize the function I(r) for r. To do that, we compute and make equal to zero its first derivative:

[tex]I^{\prime}(r)=-0.2*2r+30=0.[/tex]

Solving for r, we get:

[tex]\begin{gathered} -0.4r+30=0, \\ 0.4r=30, \\ r=\frac{30}{0.4}=75. \end{gathered}[/tex]

We have found that the maximum income is achieved when the # of price reductions is equal to r = 75.

(2) s

[tex]I(75)=-0.2*75^2+30*75+5000=6125.[/tex]

We have found that the maximum possible income per day is $6125.

(3) s

[tex]p(r)=5-0.02*75=3.5.[/tex]

We have found that the price per pack that maximizes the income is $3.5.

(4) s

[tex]1000*\text{ \$}5=\text{ \$}5000.[/tex]

With this new price structure, the company wins $6125 s

Answer

• The maximum possible income per day is $6125.

,

• The price per pack that maximizes the income is $3.5.

,

• The company makes $1125 extra with this new price structure.


Related Questions

Find the first four terms of the binomial series for the function (1+2x)^1/2

Answers

The first four terms of the binomial series are  1, x, - x²/2 and - 1/32x³ respectively.

The binomial provided to us is [tex](1+2x)^{1/2}[/tex]

To find out the first four terms of the binomial, we shall first extend the standard binomial (1+x)^n.

(1+x)^n = 1 + nx + [n(n - 1)/2!] x² +  [n(n - 1)(n - 2)/3!] x³ +...

As we can see here,

The value of x = 2x,

The value of n = 1/2.

We get,

(1+2x)^1/2 = 1 + 1/2(2x) + [1/2(1/2-1)/2!]x² + [1/2(1/2-1)(1/2-2)/3!]x³ +

(1+2x)^1/2 = 1 + x  - x²/2 - 9/4x³ + .....

From the expansion, we can see,

First term = 1

Second term = x

Third term = -x²/2

Fourth term = -1/32x³.

To know more about Binomial series, visit,

https://brainly.com/question/14004514

#SPJ1

4.
908 ÷ 10 Power of 5
0.908
90.800,000
9.08
0.00908

Answers

Answer:

0.00908 is the answer...

59 + 43.6 estimate the sum or difference

Answers

Answer:

Step-by-step explanation:

59+ 43.6

43.6=44.0

59+44

59+43.6

=103.0

4 = 4(k − 15) hhhhhhhhhhhhhhhhhhhhhhhhhhhhhhhhhhhhh

Answers

Answer:

distribute the 4 to the k and -15 so you get 4=4k-60 then add the 60 to the other 4 so its 64=4k then divide 4 to 64 and your answer is 16

Step-by-step explanation:

[tex]4=4k-60\\4k-60=4\\4k=4+60\\4k=64\\\frac{4k}{4} =\frac{64}{4} \\k=16[/tex]

As solved above k=16

In a lottery game, a player picks six numbers from 1 to 21. if the player matches all six numbers they win 50,000 dollars otherwise they lose $1

Answers

Your best option is to simply choose six distinct numbers, in any sequence. You would win if those six numbers matched the six that the lottery officials chose (again, in any sequence).

The likelihood of selecting the correct six numbers is one in

[tex]\frac{23}{6}[/tex] = [tex]\frac{23}{6(23-6)}[/tex] = [tex]\frac{23.22.21.20.19.18}{1.2.3.4.5.6}[/tex] = 100974

Therefore, 100946/100947, or a probability of losing, is very near to 1.

The total of your payouts multiplied by the probability that each scenario will occur is the anticipated value of the game. which is

(50,000) [tex]\frac{1}{100947}[/tex] + (-1) [tex]\frac{100946}{100947}[/tex] = [tex]\frac{50000-100946}{100947}[/tex] = -0.5046

You will typically break even if its value is 0. You should play the game if it is more than zero because you will typically win, and you shouldn't play it if it is less than zero because you will typically lose. The payoff is decently substantial for this, however the typical player loses just over 70 cents per round. Almost every player loses a complete dollar.

To know more about probability visit:-

https://brainly.com/question/14210034

#SPJ13

If y = 2cos(lnx). + 3sin(Inx) then Select one: 0 dy a 22 dy +r +y = 0 dx dx2 d'y x2 dy +2 dr2 dr y=0 o x2d²y dy dx + y To dx2 day 22 dy dr -y=0 dr2

Answers

Solution

Given

[tex]y=2\cos(\ln x)+3\sin(\ln x)[/tex][tex]\begin{gathered} \frac{dy}{dx}=\frac{-2}{x}\sin(\ln x)+\frac{3}{x}\cos(\ln x) \\ \\ \end{gathered}[/tex]

Determine a bank account balance if the account starts with $300 has a annual rate of %4 what is the money in the account after 10 years

Answers

Answer:420

Step-by-step explanation: multiply 300 times 4% =12. Then multiply 12 times 10=120. Then add 300 and 120=420

a gear reduction drive has three separate stages with ratio of 3:1,2.38:1 and 4.2:1. the total ratio of the drive is? ​

Answers

The total ratio of the drive is 9.58:1.

What are ratios?A relationship between two quantities that are typically stated as the product of their respective quotients is known as a ratio.The ratio formula is given as a:b = a/b for any two quantities, let's say a and b.

Given ratios of the three separate stages of a gear reduction drive are 3:1,2.38:1 and 4.2:1.

To find the total ratio of the drive we have to add the respective ratios.

When the denominator of the ratios are same, as in this case we just have to add the numerators to obtain the result.

Total ratio of the drive = 3:1 + 2.38:1 + 4.2:1 = 9.58:1

So, the total ratio of the drive is 9.58:1.

To learn more about ratios visit:

https://brainly.com/question/24199409

#SPJ9

(-a^4b³)^-4x(a^-3b^4)^5

Answers

Answer:

5

Step-by-step explanation:

use math-way

Write an expression that is equivalent to 8 using each of the following numbers and symbols once in the expression.

Answers

optionsWe want to obtain an expression equivalent to 8 using these symbols and numbers.

The option that exactly matches this is,

[tex](7\text{ }\frac{.}{.}7)^2+7[/tex]

That is because,

7 divided by 7 is 1.

1 squared is still one, and;

1 added to 7 is 8.

So one answer is option D.

We also have another answer.

Which is;

[tex](7+7^2)\frac{.}{.}7[/tex]

That's because 7 squared is 49, and 7 + 49 is 56.

56 divided by 7 is 8.

So our answers are option C and option D

Jeremy invests $8,500 into an account with a 2.4% interest rate that is compounded quarterly. How much money will be in this account after 6 years?

Round your answer to the nearest cent. Do NOT round until you have calculated the final answer.

Answers

Answer:

9812.29

Step-by-step explanation:

Using the compound interest formula,

[tex]8500 \left(1+\frac{0.024}{4} \right)^{(6)(4)} \approx 9812.29[/tex]

You have a combination lock
with 3 secret digits. Find the 3
correct digits using the
following clues and enter your
answer below.
127 One digit is right and in its place
138 One digit is right, but in the wrong place
791
Two digits are right but both are in the
wrong place
452 All digits are wrong
8
529 One digit is right, but in the wrong place

Answers

Answer:

  987

Step-by-step explanation:

You want to determine a three-digit combination based on clues about which digits are right and/or correctly placed.

Clue 1

This tells you one of digits 1, 2, or 7 is correct and correctly placed.

Clue 2 tells you that digit is not 1. Clue 4 tells you that digit is not 2.

  7 is correctly placed as the 3rd digit

Clue 2

We already know that 1 is not the correct digit. This means 3 must be the first digit, or 8 must be the first or second digit.

Clue 3

We already know that 7 is a correct digit, properly placed as the 3rd digit. Since 1 is not a correct digit, 9 must be a correct digit, properly placed as the 1st digit.

  9 is correctly placed as the 1st digit

Combining this fact with Clue 2 means ...

  8 is correctly placed as the 2nd digit

The combination is 987.

__

Check

Clues 4 and 5 are consistent with this combination.

I need help with this question on my assignment please and thank you

Answers

Given:

The expression is,

[tex]45x^2y^2+18x^3[/tex]

Simplify the expression,

[tex]\begin{gathered} 45x^2y^2+18x^3=9\times5x^2y^2+9\times2x^3 \\ =(9\times5\times x^2\times y^2)+(9\times2\times x^2\times x) \\ \text{Take 9x}^2\text{ common from both the brackets,} \\ =9\times x^2\lbrack5\times y^2\rbrack+9\times x^2\lbrack2x\rbrack \\ =9x^2(5y^2+2x) \end{gathered}[/tex]

Answer:

[tex]9x^2(5y^2+2x)[/tex]

help meeeeeeeeeeeeeeeeeeeeeeeeeeeeeeeeeee

Answers

Answer:

74.52

Step-by-step explanation:

[tex]P(10)=0.018(10)^3-0.294(10)^2+3.074(10)+55.180 \\ \\ =74.52[/tex]

Pls help with this !!!

Answers

The given graph shown in the image is a function, Option b is correct.

Given that,
A graph is shown, and we have to determine whether the graph shown is a function or not.

what is an exponential function?

The function which is in format f(x) =a^x  where a is constant and x is variable,  the domain of this exponential function lies   (-∞, ∞).

here,
The graph shows of a curve representing an exponential function,

There is why the graph shown is a function, and also the graph is continuous and as well as differentiable.

Thus, the given graph shown in the image is a function, Option b is correct.

learn more about exponential function here:

brainly.com/question/15352175

#SPJ5

T 4 Account 1 point Which trig function should Sharlot use to find the measure of angle A ? C Pashboard 5 Courses 1 1 A B 12 Calendar 2 O sine 49 cosine Inbox 3 tangent O History 4 pythagorean theorem o

Answers

In the given triangle we have :

Base line with angle A, and the Hypotenuse of angle A

Since the ratio of Base to the hypotenuse is the Cosine trignometric ratio:

[tex]\text{Cos}\theta=\frac{Base}{Hypotenuse}[/tex]

Answer : Cosine

Translate this sentence into an equation.
Carlos's height decreased by 7 is 62
Use the variable c to represent Carlos's height

Answers

c - 7 = 62. you subtract carlos’ height, “c” by 7, because it says decreased.

The functions f and g are defined as follows.
f (x) = 4x - 3 g (x) = -3x^2 - 2
Find f (-4) and g (3)/
Simplify your answers as much as possible.

Answers

Answer: [tex]f(-4)=-19, g(3)=-29[/tex]

Step-by-step explanation:

[tex]f(-4)=4(-4)-3=-19\\\\g(3)=-3(3)^2 -2=-29[/tex]

What is the value of x9+(12−y), when x = 2 and y = –5?

Answers

Answer: 35

Step-by-step explanation:

1) Substitute the x's and y's

(2)9+(12−(-5))

2) Simplify the numbers inside the equation

18+(12-(-5)

= 18+(17)

= 35

3) Solved.

35

What is (-5,2); m=2/5 in standard form?

Answers

The standard form of the straight line is given by 5y - 2x = 20 .

Ax + By = C is the standard form for two-variable linear equations.

Standard form is used, for instance, in the linear equation 6x+5y=11. This method makes finding an equation's two intercepts very simple (x and y) .Y = mx + c is the cartesian equation for a straight line, where m denotes the gradient of the line and c denotes the location of its y-axis intersection.An infinitely long object without any width, depth, or curvature is a line. Since they can exist in two, three, or higher dimensions, lines are one-dimensional objects.

We know that when a straight line passes through (a,b) with slope m then the equation of the line is given by:

y - b = m ( x - a )

putting the values in the respective places we get:

y - 2 = 2/5 × ( x + 5 )

or, 5y - 10 = 2x + 10

or , 5y - 2x = 20

Hence the standard form of the straight line is given by

5y - 2x = 20 .

To learn more about straight lines visit:

https://brainly.com/question/21627259

#SPJ1

If 1 x 10^a + 2 x 10^b + 3 x 10^c + 4 x 10^d = 24130 and a≠b≠c≠d, then what does a/2 + b/4 + c/8 + d/16 equal?

Answers

Question 1

Using scientific notation, we know a=2, b=4, c=1, and d=3. So,

[tex]\frac{a}{2}+\frac{b}{4}+\frac{c}{8}+\frac{d}{16}=1+1+\frac{1}{8}+\frac{3}{16}=37/16[/tex]

Question 2

[tex]a+13b=3(3a-b)\\\\a+13b=9a-3b\\\\16b=8a\\\\a=2b\\\\\therefore \frac{a^3}{b^3}=\frac{(2b)^3}{b^3}=\frac{8b^3}{b^3}=8[/tex]


A committee has fourteen members. There are two members that currently serve as the board's chairman
and ranking member. Each member is equally likely to serve in any of the positions. Two members are
randomly selected and assigned to be the new chairman and ranking member. What is the probability of
randomly selecting the two members who currently hold the positions of chairman and ranking member and
reassigning them to their current positions?
...

Answers

Probability of randomly selecting two members who currently hold positions of chairman and ranking member and reassigning them to their current positions from a committee of 14 members is (1 /182).

As given,

Total members in the committee=14

Number of members serve as chairman and ranking member=2

Total number of ways to assigned new position

=¹⁴P₂

=(14!) / (14-2)!

=(14 × 13 × 12!) / 12!

=14 ×13

=182

Each member is equally likely to serve in any of positions.

Each position there is only one way to reassign position.

Probability of randomly selecting two members = 1 /182

Therefore, probability of randomly selecting two members who currently hold positions of chairman and ranking member and reassigning them to their current positions from a committee of 14 members is (1 /182).

Learn more about probability here

brainly.com/question/11234923

#SPJ1

A restaurant serves an "all you can eat" buffet. At the beginning of serve, the value of the inventory is determined to be $1,200, and at the end of the meal period, it is
$800. That meal period the restaurant served 275 customers. The average food cost per customer for that meal period would be
O $3.64.
O $1.45
O $7.27.
O $5.82.

Answers

Answer:

1.45

Step-by-step explanation:

1200-800=400

400/275=1.4545454545...

the answer $1.45

1&4/5 on a number line

Answers

We can begin by converting 1 4/5 to a decimal. As a decimal, 1 4/5 converts to 1.8 or 1.80. On a number line, 1 4/5 would be between 1 and 2 whole, but swayed further toward 2.

Identify the number as real, complex, pure imaginary, or nonreal complex. (More than one of these descriptions may
apply.)
square root of -9
Select all descriptions that may apply.
A. Nonreal complex
B. Real
C. Pure imaginary
D. Complex

Answers

Answer:

acxdddhdhxhhh sax skskskkkkkddffffffrrkel sssddjeeeeerrjrrjtjjj

Question 6
Solve. Write it on paper to solve it.
5y + 4 = 4y + 5

Answers

Answer: y=1

Step-by-step explanation:

1. Subtract 4y from both sides to get y + 4 = 5.

2. Then subtract 4 from both sides to get y = 1.

Therefore, you get your answer, y = 1. Hope this helps!

after school, a group of students decides to have a snowball fight. with 2 students on a team, they can make 6 snowballs per minute. with 10 students on a team, they can make 30 snowballs per minute. what is the slope in this situation?

Answers

Based on the calculations using the given points, the slope in this situation is equal to 5.

What is a slope?

A slope is also referred to as gradient and it's typically used to describe both the ratio, direction and steepness of the function of a straight line.

How to calculate the slope of a line?

Mathematically, the slope of any straight line can be calculated by using this formula;

[tex]Slope = \frac{Change\;in\;y\;axis}{Change\;in\;x\;axis}\\\\Slope = \frac{y_2\;-\;y_1}{x_2\;-\;x_1}[/tex]

Substituting the given points into the formula, we have;

Slope = (30 - 10)/6 - 2)

Slope = 20/4

Slope = 5.

In conclusion, the slope of the line that passes through the given points is equal to 5.

Read more on slope here: brainly.com/question/3493733

#SPJ1

how do i do this problem.

Answers

ANSWER

[tex]g(5x)=75x^2\text{ + 5}[/tex]

STEP-BY-STEP EXPLANATION:

Given information

[tex]g(x)=3x^2\text{ + 5}[/tex]

Find g(5x)?

Step 1: make x = 5x

[tex]\begin{gathered} g(x)=3x^2\text{ + 5} \\ g(5x)=3(5x)^2\text{ + 5} \\ g(5x)=3(25x^2)+5^{} \\ g(5x)=75x^2\text{ + 5} \end{gathered}[/tex]

equation of the line that passes through the point (1, -1) & is parallel to the graph of y = 3/5x + 2?

Answers

Equation is 5y = 3x + 8.

What is slope?

Slope is the inclination of a line with respect to the horizontal is measured numerically. The slope of any line, ray, or line segment in analytical geometry is defined as the ratio of the vertical to the horizontal distance between any two points on the line, ray, or segment. The ratio of the increase in elevation between two points to the run in elevation between those same two points is referred to as the slope.

Given Data

line that passes through the point (1, -1) &

is parallel to the graph of y = 3/5x + 2

y = mx + c

y = [tex]\frac{3}{5}[/tex]x + 2

m = [tex]\frac{3}{5}[/tex]

Equating points,

-1 = [tex]\frac{3}{5}[/tex](1) + c

-1 = [tex]\frac{3}{5}[/tex] + c

-1 - [tex]\frac{3}{5}[/tex] = c

[tex]\frac{-5-3}{5}[/tex] = c

[tex]\frac{-8}{5}[/tex]

Equation-

y = [tex]\frac{3}{5}[/tex]x + [tex]\frac{8}{5}[/tex]

5y = 3x + 8

Equation is 5y = 3x + 8.

To learn more about slope, visit:

https://brainly.com/question/3605446

#SPJ9

The Pythagorean Identity is sin^2x+cos^2x=1. Write this equation two other ways.​

Answers

[tex]\sin^2 x=1-\cos^2 x \\ \\ \cos^2 x=1-\sin^2 x[/tex]

Other Questions
Name and explain 3 fad diets Determine numerical values for each of the three mesh currents as labeled inthe circuit diagram of Fig. 4.58. Question 9(Multiple Choice Worth 1 points) (04.05 MC) A company determines an employee's starting salary according to the number of years of experience, as detailed in the table. Years of experience Salary 0 $40,000 1 $42,150 $44,260 $46,785 $48,820 $51,126 2 3 4 5 Use the equation for the line of best fit to predict the salary for an employee with 7 years of experience? (Round your answer to the nearest dollar) O $52,900 O $53,340 O $53,914 O $55,573 Please help Ill mark you as brainliest if correct!! PLSSS HELP MEEE WILL GIVE 20 POINTSEstimate the quotient of 42.4 7.28. 9 8 7 6 What is the difference between old agriculture and new agriculture? PLEASE HELP WITH THIS QUESTION!!!!!!!!!! Select the correct answer. A soccer team wins 65% of its matches, and 15% of its matches end in a draw. If the team is scheduled to play 20 matches, about how many matches is it expected to lose? A. 13 B. 4 C. 8 D. 1 Reset its B.4 Find the value -2+5/3 100 points !!!!!!!!!! What amount of pure acid must be added to 400 mL of a 30% acid solution to produce a 75% acid solution? A yoga studio charges a $140 yearly fee for membership plus $15 for each class. What is the average total cost per class if a studio member takes 50 classes per year? angle A and angle B are vertical angles. If mA = (2x +29) and mB = (3x - 17), then find the measure of angle B. A student is running for president of student government. Her friend has designed a campaign poster that measures 27 feet by 14 feet. She duplicated the poster on arectangular button measuring 6 inches by 3 1/9 inches. What is the scale factor? Find the slope of the graph What is a pronoun? A: a word that has a positive connotation B: a word used by professionals in a specific field C: a word used to replace a proper noun D: a word used to qualify the meaning of another word If the location of A (2, -5)with reference to point (0,0)What is the direction of A I NEED HELP ASAPWhat is the main idea presented in this passage?[In about 1200, a] major population shift saw [the AncestralPuebloans] begin to move back into the cliff alcoves [orrecesses] that sheltered their ancestors centuries before.Why did they make this move? We don't know. Perhaps itwas for defense; perhaps it was for religious orpsychological reasons; perhaps alcoves offered betterprotection from the elements. Whatever the reason, orreasons, it gave rise to the cliff dwellings for which MesaVerde is most famous.1OA. Mesa Verde is famous for its Ancestral Puebloan cliff dwellings.A. Mess Verde is famous for its Ancestral Puebloan cliff dwellings.B. The Ancestral Puebloan population shifted many times throughout history.C. Cliff dwellings made it easier for the Ancestral Puebloan to defend themselves.D. The Ancestral Puebloan moved into cliff dwellings for unknown reasons. Earning Money: Marcus works at an ice cream shop. He eamns $14.50 eachhour. Use Marcus's timesheet for the week to answer the questionsDate. Time in. Time outApril 11 8:45 am 12.00pmApril 12 9:00am 12.00pmApril 13 7:30am 12:00pmApril 14 8:00am 1:15pmApril 15 8:30am 11:15amA. How many hours did Marcus work for the week?B. How much money did Marucs earn for the week? Cave artists took advantage of the rough stone surfaces of the cave. In some caves, such as the Chauvet Cave in France, artists used the bumps in the stone to emphasize parts of the animal. In the Chauvet Cave, an artist painted a bears paw over a knob or bulge in the stone, which makes the paw stand out and look fiercer. Reflect on the elements of art. What element was this cave artist demonstrating? (1 point) Responses A.texture B.perspective C. value D. color